$sigma$-finite measure $mu$ so that $L^p(mu) subsetneq L^q(mu)$ (proper subset)











up vote
0
down vote

favorite












I'm looking for a $sigma$-finite measure $mu$ and a measure space so that for



$1 le p <q le infty$



$$L^p(mu) subsetneq L^q(mu)$$



I tried the following:



Let $1 le p <q le infty$ and $lambda$ the Lebesgue measure on $(1,infty)$ which is $sigma$-finite.



$x^alpha$ is integrable on $(1,infty) Leftrightarrow alpha <-1$.



Choose $b$ so that $1/q<b<1/p Leftrightarrow -bq<-1, -bp>-1$.



Then $x^{-b}chi_{(1,infty)}$$in L^q$ but $notin L^p$ because $x^{-bq}$ is integrable because the exponent $-bq<-1$ and $x^{-bp}$ isn't integrable because the exponent $-bp>-1$. Now I found a function that is in $L^p$ but not in $L^q$. But that doesn't really show that $L^p subsetneq L^q$, meaning $L^p$ is a proper subset of $L^q$, right (because I don't know if every element of $L^p$ is also an element of $L^q$)?



Thanks in advance!










share|cite|improve this question


























    up vote
    0
    down vote

    favorite












    I'm looking for a $sigma$-finite measure $mu$ and a measure space so that for



    $1 le p <q le infty$



    $$L^p(mu) subsetneq L^q(mu)$$



    I tried the following:



    Let $1 le p <q le infty$ and $lambda$ the Lebesgue measure on $(1,infty)$ which is $sigma$-finite.



    $x^alpha$ is integrable on $(1,infty) Leftrightarrow alpha <-1$.



    Choose $b$ so that $1/q<b<1/p Leftrightarrow -bq<-1, -bp>-1$.



    Then $x^{-b}chi_{(1,infty)}$$in L^q$ but $notin L^p$ because $x^{-bq}$ is integrable because the exponent $-bq<-1$ and $x^{-bp}$ isn't integrable because the exponent $-bp>-1$. Now I found a function that is in $L^p$ but not in $L^q$. But that doesn't really show that $L^p subsetneq L^q$, meaning $L^p$ is a proper subset of $L^q$, right (because I don't know if every element of $L^p$ is also an element of $L^q$)?



    Thanks in advance!










    share|cite|improve this question
























      up vote
      0
      down vote

      favorite









      up vote
      0
      down vote

      favorite











      I'm looking for a $sigma$-finite measure $mu$ and a measure space so that for



      $1 le p <q le infty$



      $$L^p(mu) subsetneq L^q(mu)$$



      I tried the following:



      Let $1 le p <q le infty$ and $lambda$ the Lebesgue measure on $(1,infty)$ which is $sigma$-finite.



      $x^alpha$ is integrable on $(1,infty) Leftrightarrow alpha <-1$.



      Choose $b$ so that $1/q<b<1/p Leftrightarrow -bq<-1, -bp>-1$.



      Then $x^{-b}chi_{(1,infty)}$$in L^q$ but $notin L^p$ because $x^{-bq}$ is integrable because the exponent $-bq<-1$ and $x^{-bp}$ isn't integrable because the exponent $-bp>-1$. Now I found a function that is in $L^p$ but not in $L^q$. But that doesn't really show that $L^p subsetneq L^q$, meaning $L^p$ is a proper subset of $L^q$, right (because I don't know if every element of $L^p$ is also an element of $L^q$)?



      Thanks in advance!










      share|cite|improve this question













      I'm looking for a $sigma$-finite measure $mu$ and a measure space so that for



      $1 le p <q le infty$



      $$L^p(mu) subsetneq L^q(mu)$$



      I tried the following:



      Let $1 le p <q le infty$ and $lambda$ the Lebesgue measure on $(1,infty)$ which is $sigma$-finite.



      $x^alpha$ is integrable on $(1,infty) Leftrightarrow alpha <-1$.



      Choose $b$ so that $1/q<b<1/p Leftrightarrow -bq<-1, -bp>-1$.



      Then $x^{-b}chi_{(1,infty)}$$in L^q$ but $notin L^p$ because $x^{-bq}$ is integrable because the exponent $-bq<-1$ and $x^{-bp}$ isn't integrable because the exponent $-bp>-1$. Now I found a function that is in $L^p$ but not in $L^q$. But that doesn't really show that $L^p subsetneq L^q$, meaning $L^p$ is a proper subset of $L^q$, right (because I don't know if every element of $L^p$ is also an element of $L^q$)?



      Thanks in advance!







      real-analysis analysis measure-theory






      share|cite|improve this question













      share|cite|improve this question











      share|cite|improve this question




      share|cite|improve this question










      asked yesterday









      user610431

      456




      456






















          1 Answer
          1






          active

          oldest

          votes

















          up vote
          0
          down vote













          The inclusion $L^p(mu) subset L^q(mu)$ for $1leq p < q leq +infty$ holds if and only if
          $$
          inf{mu(A): Ainmathcal{M}, mu(A)>0} > 0.
          $$

          Hence your $mu$ must necessarily be an atomic measure.



          For example, you can consider the spaces $ell^p$. More precisely, you equip $mathbb{N}$ with the counting measure, so that
          $$
          ell^p := {x = (x_1, x_2, ldots): sum_{j=1}^infty |x_j|^p < infty}.
          $$






          share|cite|improve this answer























          • Thanks, but unfortunately I don't really understand what exactly you mean. Can you give me an example of such an measure?
            – user610431
            yesterday










          • Done. (Edit in the main text.)
            – Rigel
            14 hours ago











          Your Answer





          StackExchange.ifUsing("editor", function () {
          return StackExchange.using("mathjaxEditing", function () {
          StackExchange.MarkdownEditor.creationCallbacks.add(function (editor, postfix) {
          StackExchange.mathjaxEditing.prepareWmdForMathJax(editor, postfix, [["$", "$"], ["\\(","\\)"]]);
          });
          });
          }, "mathjax-editing");

          StackExchange.ready(function() {
          var channelOptions = {
          tags: "".split(" "),
          id: "69"
          };
          initTagRenderer("".split(" "), "".split(" "), channelOptions);

          StackExchange.using("externalEditor", function() {
          // Have to fire editor after snippets, if snippets enabled
          if (StackExchange.settings.snippets.snippetsEnabled) {
          StackExchange.using("snippets", function() {
          createEditor();
          });
          }
          else {
          createEditor();
          }
          });

          function createEditor() {
          StackExchange.prepareEditor({
          heartbeatType: 'answer',
          convertImagesToLinks: true,
          noModals: true,
          showLowRepImageUploadWarning: true,
          reputationToPostImages: 10,
          bindNavPrevention: true,
          postfix: "",
          imageUploader: {
          brandingHtml: "Powered by u003ca class="icon-imgur-white" href="https://imgur.com/"u003eu003c/au003e",
          contentPolicyHtml: "User contributions licensed under u003ca href="https://creativecommons.org/licenses/by-sa/3.0/"u003ecc by-sa 3.0 with attribution requiredu003c/au003e u003ca href="https://stackoverflow.com/legal/content-policy"u003e(content policy)u003c/au003e",
          allowUrls: true
          },
          noCode: true, onDemand: true,
          discardSelector: ".discard-answer"
          ,immediatelyShowMarkdownHelp:true
          });


          }
          });














           

          draft saved


          draft discarded


















          StackExchange.ready(
          function () {
          StackExchange.openid.initPostLogin('.new-post-login', 'https%3a%2f%2fmath.stackexchange.com%2fquestions%2f2996992%2fsigma-finite-measure-mu-so-that-lp-mu-subsetneq-lq-mu-proper-sub%23new-answer', 'question_page');
          }
          );

          Post as a guest
































          1 Answer
          1






          active

          oldest

          votes








          1 Answer
          1






          active

          oldest

          votes









          active

          oldest

          votes






          active

          oldest

          votes








          up vote
          0
          down vote













          The inclusion $L^p(mu) subset L^q(mu)$ for $1leq p < q leq +infty$ holds if and only if
          $$
          inf{mu(A): Ainmathcal{M}, mu(A)>0} > 0.
          $$

          Hence your $mu$ must necessarily be an atomic measure.



          For example, you can consider the spaces $ell^p$. More precisely, you equip $mathbb{N}$ with the counting measure, so that
          $$
          ell^p := {x = (x_1, x_2, ldots): sum_{j=1}^infty |x_j|^p < infty}.
          $$






          share|cite|improve this answer























          • Thanks, but unfortunately I don't really understand what exactly you mean. Can you give me an example of such an measure?
            – user610431
            yesterday










          • Done. (Edit in the main text.)
            – Rigel
            14 hours ago















          up vote
          0
          down vote













          The inclusion $L^p(mu) subset L^q(mu)$ for $1leq p < q leq +infty$ holds if and only if
          $$
          inf{mu(A): Ainmathcal{M}, mu(A)>0} > 0.
          $$

          Hence your $mu$ must necessarily be an atomic measure.



          For example, you can consider the spaces $ell^p$. More precisely, you equip $mathbb{N}$ with the counting measure, so that
          $$
          ell^p := {x = (x_1, x_2, ldots): sum_{j=1}^infty |x_j|^p < infty}.
          $$






          share|cite|improve this answer























          • Thanks, but unfortunately I don't really understand what exactly you mean. Can you give me an example of such an measure?
            – user610431
            yesterday










          • Done. (Edit in the main text.)
            – Rigel
            14 hours ago













          up vote
          0
          down vote










          up vote
          0
          down vote









          The inclusion $L^p(mu) subset L^q(mu)$ for $1leq p < q leq +infty$ holds if and only if
          $$
          inf{mu(A): Ainmathcal{M}, mu(A)>0} > 0.
          $$

          Hence your $mu$ must necessarily be an atomic measure.



          For example, you can consider the spaces $ell^p$. More precisely, you equip $mathbb{N}$ with the counting measure, so that
          $$
          ell^p := {x = (x_1, x_2, ldots): sum_{j=1}^infty |x_j|^p < infty}.
          $$






          share|cite|improve this answer














          The inclusion $L^p(mu) subset L^q(mu)$ for $1leq p < q leq +infty$ holds if and only if
          $$
          inf{mu(A): Ainmathcal{M}, mu(A)>0} > 0.
          $$

          Hence your $mu$ must necessarily be an atomic measure.



          For example, you can consider the spaces $ell^p$. More precisely, you equip $mathbb{N}$ with the counting measure, so that
          $$
          ell^p := {x = (x_1, x_2, ldots): sum_{j=1}^infty |x_j|^p < infty}.
          $$







          share|cite|improve this answer














          share|cite|improve this answer



          share|cite|improve this answer








          edited 14 hours ago

























          answered yesterday









          Rigel

          10.6k11319




          10.6k11319












          • Thanks, but unfortunately I don't really understand what exactly you mean. Can you give me an example of such an measure?
            – user610431
            yesterday










          • Done. (Edit in the main text.)
            – Rigel
            14 hours ago


















          • Thanks, but unfortunately I don't really understand what exactly you mean. Can you give me an example of such an measure?
            – user610431
            yesterday










          • Done. (Edit in the main text.)
            – Rigel
            14 hours ago
















          Thanks, but unfortunately I don't really understand what exactly you mean. Can you give me an example of such an measure?
          – user610431
          yesterday




          Thanks, but unfortunately I don't really understand what exactly you mean. Can you give me an example of such an measure?
          – user610431
          yesterday












          Done. (Edit in the main text.)
          – Rigel
          14 hours ago




          Done. (Edit in the main text.)
          – Rigel
          14 hours ago


















           

          draft saved


          draft discarded



















































           


          draft saved


          draft discarded














          StackExchange.ready(
          function () {
          StackExchange.openid.initPostLogin('.new-post-login', 'https%3a%2f%2fmath.stackexchange.com%2fquestions%2f2996992%2fsigma-finite-measure-mu-so-that-lp-mu-subsetneq-lq-mu-proper-sub%23new-answer', 'question_page');
          }
          );

          Post as a guest




















































































          Popular posts from this blog

          Plaza Victoria

          In PowerPoint, is there a keyboard shortcut for bulleted / numbered list?

          How to put 3 figures in Latex with 2 figures side by side and 1 below these side by side images but in...